OAE 090

Pataasin ang iyong marka sa homework at exams ngayon gamit ang Quizwiz!

A syllable must contain: a) A vowel b) A consonant c) Both a vowel and a consonant d) A meaning

a) A vowel. A syllable is a minimal sound unit arranged around a vowel. For example, academic has four syllables: a/ca/dem/ic. It is possible for a syllable to be a single vowel, as in the above example. It is not possible for a syllable to be a single consonant.

The kindergarten teacher is concerned about three of her students. While they are enthusiastic about writing, they do not always recognize letters, confusing b, d, and p, or e and o. They do, however, know which sounds go with certain letters when they are orally drilled. When they write, they appear to be attempting letter-sound associations. "Now I'm writing M," the teacher heard one boy say as he scripted a large N in the upper right corner of his paper. He studied it for a moment and added, "Nope, it needs another leg." The student then wrote an I beside the N. "There," he said, "Now you are an M. I can write the word 'man,' because now I have M." The child then moved to the lower left corner of the paper. "M-A-N," he said to himself, slowly pronouncing each sound. "I already have that M. Here is where the rest of the word goes." He turned the paper sideways and wrote N. The second child sang to herself as she gripped the crayon and scribbled lines here and there on her paper. Some of the lines resembled letters, but few actually were. Others were scribbles. As she "wrote," she seemed to be making up a story and seemed to believe she was writing the story down. The third child didn't vocalize at all while he worked. He gripped the paper and carefully wrote the same letter over and over and over and over. Sometimes the letter was large, sometimes tiny. He turned the paper in every direction so that sometimes the letter was sideways or upside down. Sometimes he flipped it backward. "What are you writing?" the teacher asked him. "My name," the child told her. The teacher then realized the letter was, indeed, the first letter of his name. She gently told him he had done a fine job of writing the first letter of his name. Did he want her to help him write the rest of it? "Nope," he cheerfully told her, "it's all here." He pointed at one of the letters and "read" his full name. He pointed at another letter and again seemed to believe it represented all the sounds of his name. In the above example, the emergent writers are demonstrating their understanding that letters symbolize predictable sounds, that words begin with an initial sound/letter, and that by "writing," they are empowering themselves by offering a reader access to their thoughts and ideas. The next three stages the emergent writers will pass through in order will most likely be: a) Scripting the end-sound to a word (KT=cat); leaving space between words; writing from the top left to the top right of the page, and from top to bottom b) Scripting the end-sound to a word (KT=cat); writing from the top left to the top right of the page, and from top to bottom; separating the words from one another with a space between c) Leaving space between the initial letters that represent words; writing from the top left to the top right of the page, and from top to bottom; scripting the final sound of each word as well as the initial sound (KT=cat) d) Drawing a picture beside each of the initial sounds to represent the entire word; scripting the end-sound to a word (KT=cat); scripting the interior sounds that compose the entire word (KAT=cat)

a) Scripting the end-sound to a word (KT=cat); leaving space between words; writing from the top left to the top right of the page, and from top to bottom. Each of these steps is progressively more abstract. Scripting the end-sound to a word helps a young writer recognize that words have beginnings and endings. This naturally leads to the willingness to separate words with white space so that they stand as individual entities. Once this step is reached, the child realizes that in English, writing progresses from left to right and from the top of the page to the bottom.

The kindergarten teacher is concerned about three of her students. While they are enthusiastic about writing, they do not always recognize letters, confusing b, d, and p, or e and o. They do, however, know which sounds go with certain letters when they are orally drilled. When they write, they appear to be attempting letter-sound associations. "Now I'm writing M," the teacher heard one boy say as he scripted a large N in the upper right corner of his paper. He studied it for a moment and added, "Nope, it needs another leg." The student then wrote an I beside the N. "There," he said, "Now you are an M. I can write the word 'man,' because now I have M." The child then moved to the lower left corner of the paper. "M-A-N," he said to himself, slowly pronouncing each sound. "I already have that M. Here is where the rest of the word goes." He turned the paper sideways and wrote N. The second child sang to herself as she gripped the crayon and scribbled lines here and there on her paper. Some of the lines resembled letters, but few actually were. Others were scribbles. As she "wrote," she seemed to be making up a story and seemed to believe she was writing the story down. The third child didn't vocalize at all while he worked. He gripped the paper and carefully wrote the same letter over and over and over and over. Sometimes the letter was large, sometimes tiny. He turned the paper in every direction so that sometimes the letter was sideways or upside down. Sometimes he flipped it backward. "What are you writing?" the teacher asked him. "My name," the child told her. The teacher then realized the letter was, indeed, the first letter of his name. She gently told him he had done a fine job of writing the first letter of his name. Did he want her to help him write the rest of it? "Nope," he cheerfully told her, "it's all here." He pointed at one of the letters and "read" his full name. He pointed at another letter and again seemed to believe it represented all the sounds of his name. At what point should the kindergarten teacher in the above example offer the three children picture books and ask them to read to her? a) When the three children are all able to script initial sounds, end sounds, and interior sounds they are ready to decode words. She should make her request at this point b) As each child reaches the stage in which he or she can script initial sounds, end sounds, and interior sounds, the teacher should ask only that child has had experience of writing in the same manner, he won't be able to make sense of the words d) The teacher should encourage all students to "read" pictures books from the first day of school. Talking about the pictures from page to page gives young readers the idea that books are arranged sequentially. Pictures also offer narrative coherence and contextual clues. Emergent readers who are encouraged to enjoy books will more readily embrace the act of reading. Holding a book and turning pages gives young readers a familiarity with them

d) The teacher should encourage all students to "read" pictures books from the first day of school. Talking about the pictures from page to page gives young readers the idea that books are arranged sequentially. Pictures also offer narrative coherence and contextual clues. Emergent readers who are encouraged to enjoy books will more readily embrace the act of reading. Holding a book and turning pages gives young readers a familiarity with them

An ORF is: a) An Oral Reading Fluency assessment b) An Occasional Reading Function assessment c) An Oscar Reynolds Feinstein assessment d) An Overt Reading Failure assessment

a) An Oral Reading Fluency assessment. ORF stands for oral reading fluency. This assessment measures the words correct per minute (WCPM) by subtracting the number of errors made from the total number of words orally read in a one- to two- minute period of time. It is used to find a student's Instructional reading level, to identify readers who are having difficulties, and to track developing fluency and word recognition over time.

Syllable types include: a) Closed, open, silent e, vowel team, vowel-r, and consonant-le b) Closed, open, silent, double-vowel, r, and le c) Closed, midway, open, emphasized, prefixed, and suffixed d) Stressed, unstressed, and silent

a) Closed, open, silent e, vowel team, vowel-r, and consonant-le. A closed syllable ends with a consonant, such as cat. Open syllables end with a vowel, such as he. Vowel team syllables contain two vowels working together, such as main. Vowel-r syllables such as er and or frequently occur as suffixes. Consonant-le syllables also typically occur as suffixes, such as battle or terrible.

A class will visit an assisted living facility to interview residents about their lives. Each group of three has selected a theme such as love, work, or personal accomplishment and written several questions around the theme. Next each group practices interviewing one another. The teacher then asks all the students to discuss the questions that caused them to respond most thoughtfully, as well as those they were less inspired by. The students decided the questions that were easiest to respond to asked for very specific information; for example, one inspiring question was, "Please tell me about something you learned to do as a child that affected the direction of your life." Those that were uninspiring were too broad, for example, "Please tell me about your happiest memory." After they interview the residents, each group of three students will work together to write a piece about the resident. This kind of approach is called: a) Collaborative learning b) Companion learning c) Bonded learning d) Group learning

a) Collaborative learning. A group of students working together on a project are applying numerous learning strategies at once. Collaborative learning is a hands-on approach that actively involves students in the learning process. Students involved in collaborative learning typically retain the lesson better.

Which of the following criteria would be most important to consider when selecting "leveled texts" for use in assessments and guided reading with beginning-level readers? A. The texts should use repeated words and natural oral language structures. B. The texts should require readers to use problem-solving to connect text to illustrations. C. The texts should emphasize use of literary language and dialogues. D. The texts should feature a range of punctuation and context-specific vocabulary.

A. Beginning readers need lots of practice reading a variety of texts written at their independent reading levels to reinforce their knowledge of phonics and sight words already taught, build their automaticity, and improve their reading fluency. Leveled texts are series of texts constructed using controlled vocabulary and syntactic structures. Within a particular level, the texts share many of the same vocabulary words and follow basic syntactic structures, both of which gradually increase in difficulty in successive levels. Matching students to appropriate-level texts is key to providing beginning readers with effective practice opportunities. B is incorrect because the primary goal of using these texts with beginning readers is to reinforce students' decoding skills and promote reading fluency, not to promote their use of picture clues. They are also not designed to promote students' knowledge of literary language or devices (C). Because of their controlled nature, leveled texts feature a limited range of punctuation, and they purposefully introduce only a few new content vocabulary words per text (D).

A sixth-grade class is working on an Internet research project about various natural resources and their uses. The teacher could best support students' effective use of the Internet for their research by: A. providing students with a checklist of questions that prompt critical evaluation of information on Web sites. B. giving students a list of Web sites that have been pre-approved based on the sites' reading levels. C. encouraging students to search for Web sites that are easy to navigate and that contain familiar vocabulary. D. teaching students to employ a variety of search engines to locate relevant Web sites.

A. Determining whether a given Web site contains reliable information is an important step in conducting research on the Internet. Having students question the validity of content they read on a Web site will build this skill, promoting their ability to conduct more effective and bias-free research. B is incorrect because giving students a list of pre-approved Web sites does not promote their independent research skills. Encouraging them to rely only on user-friendly Web sites (C) could lead students to ignore Web sites that contain valuable, objective information. Employing a variety of search engines (D) does not necessarily lead to more effective research, since different search engines are likely to identify a similar list of potential Web sites.

Electronic reading books are advantageous for beginning or struggling readers primarily because this type of computer software: A. scaffolds learning by providing a high level of interactivity. B. helps students develop familiarity with reading from a computer screen. C. provides students with models of good reading practices and habits. D. minimizes the focus on written text by using sound effects and voices to convey meaning.

A. Electronic books can offer many scaffolds to beginning or struggling readers, including providing the opportunity to hear the pronunciation or the definition of an unfamiliar word in the text or to listen to the whole text read aloud by a proficient reader. While electronic books can be available for use on a computer, they are not necessarily limited to this medium (B). Electronic books can provide good models of fluent reading; however, they are inanimate objects, so they cannot model good reading practices or habits (C). While electronic books may use auditory input to enhance comprehension (D), this is intended to be a scaffold for the reader and not a substitute for the text.

Which of the following best describes the primary advantage of having a student read a passage silently and then provide a "retelling" as a means of assessing the student's comprehension, rather than having the student answer questions? A. A retelling is open-ended and requires the student to construct a description of the passage more independently of the examiner. B. The results of a retelling are more objective and easier to quantify than the results of direct questioning. C. The procedure involved in retelling tends to be more familiar to a wider range of students, including English Language Learners. D. A retelling can provide information about the student's inferential comprehension skills, which questioning cannot provide.

A. Having a young student provide a "retelling" (or having an older student construct a summary) of a text removes clues or scaffolds from the assessment that teacher questions often provide and ensures that a student's responses are based solely on his or her reading of the text. Some disadvantages of retellings include that the results are not easy to quantify (B) and the procedure is often unfamiliar to students, particularly to students who are new to U.S. schools (C). Also, since retellings are unstructured and usually elicit only explicitly stated details about the text, this type of assessment is not necessarily effective in assessing a student's inferential comprehension skills (D).

A fifth-grade teacher plans to have students read a chapter about the American Revolutionary War from their social studies textbook. The following is an excerpt from the chapter. The Battle of Bunker Hill took place on June 17, 1775. At the time, the American army occupied the area from Cambridge to the Mystic River. American troops gathered in Cambridge Common on the evening of June 16, 1775, and set out for Bunker Hill. Upon reaching Bunker Hill, however, officers decided to move to Breed's Hill, a smaller hill closer to Boston. The teacher asks students to locate and mark places mentioned in the chapter on a map as they read. This activity is most likely to help students: A. use visualization to facilitate their comprehension of the text. B. paraphrase content to make the text more understandable. C. connect elements in the text to their background knowledge. D. identify the text's main ideas and supporting details.

A. The passage mentions several sites or landmarks with which students may be unfamiliar. Locating and marking these sites on a map would improve the students' ability to visualize the direction of the troop movements mentioned in the passage. B, C, and D are not skills that would be scaffolded, facilitated, or reinforced by having the students consult maps.

Two proficient readers are answering post-reading comprehension questions about a chapter in a content-area textbook. • The first student demonstrates exceptional recall of details from the chapter but has difficulty answering questions about the gist of the chapter. • The second student can give an outstanding summary of the chapter but has difficulty remembering specific facts from the chapter. Which of the following best explains the most likely reason for the students' varied understanding of the text? A. The first student is more proficient than the second student at using metacognitive comprehension strategies to make sense of the text. B. Each student applied different reading comprehension skills when reading the text. C. The second student is more proficient at reading for literal understanding than for inferential understanding. D. Each student brought a unique set of prior experiences to the reading of the text.

B. Many factors contribute to reading comprehension. Skilled readers use different comprehension strategies to achieve different purposes. For example, the first student described in this item may have skimmed the chapter to look for specific words or phrases mentioned in the comprehension questions, while the second student may have scanned the chapter for main ideas but did not dwell on individual terms or facts. A is incorrect because the first student had difficulty answering questions about the gist of the chapter, so it is likely that that student did not apply metacognitive strategies to enhance understanding during reading. The second student was able to construct a strong summary of the chapter, which suggests an ability to use inferential skills, so C is incorrect. While background knowledge and experiences have a strong influence on reading comprehension (D), this factor cannot explain why the students focused on such different levels of the text.

A third-grade teacher periodically reads aloud from a chapter in content-area textbooks and describes his thought processes as he reads. Following is an example: "'The moon does not shine on its own. The sun's light reflects off the moon.' Hmm. I'm imagining that the sun is like a flashlight shining on the moon in the dark. 'As the moon rotates, only the part that faces the sun is visible from the Earth.' I'm not quite sure what "visible" means, but it sounds kind of like vision, which I know has to do with eyes. It probably means the part that we can see from the Earth. Now, that makes me wonder— why do we see different amounts of the moon at different times? Let's see if the next part of the chapter explains this . . ." This practice is most likely to promote students' reading proficiency by: A. exposing them to new vocabulary in context. B. modeling for them metacognitive comprehension strategies. C. giving them an example of fluent oral reading. D. summarizing for them the main ideas of an expository text.

B. Metacognitive reading comprehension strategies prompt students to think about their thinking as they read a text. The teacher models two powerful metacognitive strategies: visualizing to support comprehension and self-questioning to clarify understanding and to set a purpose for further reading. A is incorrect because while the teacher talks about the vocabulary word visible and determines what it means deductively, the focus of this portion of the teacher's think-aloud is promoting the students' comprehension of the sentence as a whole in which the word visible occurs. C is incorrect because the teacher pauses several times during reading to make comments about the text, so the teacher does not present an example of fluent oral reading. The teacher could conceivably summarize the text at the end of the think-aloud (D), but there is no evidence in the description to suggest this will occur.

Which of the following types of assessments would best provide information about the comparative reading proficiency of students in an elementary school? A. a test of vocabulary development B. a norm-referenced survey test C. a reading miscue inventory D. a diagnostic portfolio

B. Norm-referenced tests are designed specifically for the purpose of comparing students' performance. Norms are statistics that describe the test performance of a representative sample group. In general, the tests in A and C are designed to measure the degree to which students meet specific objectives (i.e., they are criterion referenced); therefore, A and C are incorrect. A diagnostic portfolio (D) is individually administered and would most likely contain a comprehensive battery of reading assessments designed for diagnostic purposes.

A fifth-grade teacher gives students a reading guide for an informational text that they will be reading independently. The reading guide contains various activities, including prompting students to summarize certain passages, to explain relationships between concepts according to specific information in the text, and to determine the meaning of domain-specific words based on appositives or appositive phrases embedded in the text. This reading guide is likely to be most effective for achieving which of the following instructional purposes? A. developing students' ability to read the text evaluatively B. encouraging students to read and interact closely with the text C. supporting students' development of prosodic reading skills D. teaching students to adjust their reading rate based on text complexity

B. Reading guides such as the one described compel students to reread a complex academic text multiple times so they can complete the activities in the guide, which are specifically designed to help the students interact with the content and language of the text. A is incorrect because the activities do not focus on developing students' evaluative comprehension skills. C is incorrect because use of such a guide would not promote development of prosodic reading, since it is a silent reading activity that does not involve explicit or implicit teacher modeling of or instruction in prosodic reading. A text-based reading guide would not teach students how to adjust their reading rate based on text difficulty (D) because it focuses on just one specific text.

Skimming is likely to be the most effective strategy for accomplishing which of the following reading tasks? A. evaluating the validity of information on an Internet Web site B. previewing a chapter in a content-area textbook C. synthesizing information from various sources for a research report D. studying specific facts for a content- area exam

B. Skimming involves a quick, superficial reading of a text to get an overall impression of the material. This would be an appropriate and effective strategy for previewing a textbook chapter. The other responses are incorrect because evaluating the validity of information on a Web site (A), synthesizing information from a variety of sources for research purposes (C), and studying facts for a content-area exam (D) all require a more careful reading of textual materials.

As a first-grade teacher reads a big book to a group of students, the teacher points to the beginning consonants of selected words and accentuates the sound the initial letter makes. This activity is most likely to promote the students': A. awareness of multi-syllable words. B. ability to isolate individual sounds in words. C. structural analysis skills. D. ability to blend the sounds in words.

B. The activity described contributes to students' beginning reading development in a number of ways, including reinforcing their phonemic awareness (i.e., their ability to distinguish the separate phonemes in a spoken word), their knowledge of letter-sound correspondences, and their understanding of the alphabetic principle. The other responses are incorrect because the activity focuses students' attention on only the first letter of the words, so the activity would not promote students' awareness of multisyllable words (A), their structural analysis skills (C), or their ability to blend all the sounds in a word (D).

Which of the following is the most important reason for a fourth-grade teacher to assign a variety of high-quality trade books as a component of reading instruction? A. The themes typical of children's literature tend to reinforce students' development of literal comprehension skills. B. Reading across genres helps students develop an understanding of the structures and features of different texts. C. Simplified syntax and controlled vocabulary provide necessary scaffolding for students who are struggling readers. D. Reading diverse texts helps to promote students' development of phonological and phonemic awareness skills.

B. Trade books are books marketed to a general audience, as opposed to books developed for specialized audiences. Children's trade books cover a range of topics and include both fiction and nonfiction texts. Wide reading of trade books provides students with exposure to many genres and helps them begin to develop an awareness of the structural differences between genres. Trade books are not limited to specific themes (A), nor do they use simplified syntax or controlled vocabulary (C). Reading trade books does not explicitly (or implicitly) teach fourth-grade students phonological and phonemic awareness skills (D), which are typically developed at a much earlier grade level using primarily oral language activities.

An advantage of using assessment tools such as portfolios and scoring rubrics is that they: A. provide more objective results than do multiple-choice tests. B. promote student participation in self- assessment activities. C. ensure consistency among different evaluators. D. offer more reliable assessment data.

B. Typically, the development of student work portfolios involves students in selecting and self-assessing some or all of the materials that are included in their individual portfolios. Students can use scoring rubrics to self-assess and guide their work. The other responses are incorrect because students and teachers can develop their own criteria for selecting student work for a particular portfolio and can develop their own scoring rubrics for evaluating that work; thus, the results of this form of assessment are neither more objective (A), nor more consistent (C), nor more reliable (D) than the results of standardized assessments.

Considerations of validity in test construction relate most closely to: A. how a particular examinee's test performance relates to a pre-established standard. B. whether the test questions effectively measure their specified content. C. how a particular examinee's test performance compares to the performance of other examinees. D. whether the test results are likely to be repeatable with a similar examinee test group.

B. Validity is the degree to which a test measures what it is intended to measure. A is incorrect because criterion referencing is the term used when a student's performance is related to a preestablished standard or set of objectives. C describes norm referencing. D describes test reliability.

Which of the following informal assessment results provides the clearest indication that a kindergarten child has attained a beginning level of phonemic awareness? A. The student can clap the "beats" or syllables of familiar multisyllable words. B. The student can delete the second "word" or syllable in compound words. C. The student can identify the beginning sound of single-syllable words. D. The student can substitute phonemes in the medial position of single- syllable words.

C. Phonemic awareness is a specific type of phonological awareness involving the ability to distinguish the separate phonemes in a spoken word. Identifying the beginning sound of a single-syllable word is typically one of the earliest phonemic awareness skills developed, while substituting medial phonemes (D) is considered a more advanced phonemic awareness skill. A and B describe skills at the syllable or word level and are therefore not considered phonemic awareness skills.

Which of the following types of activities would be most important to include on a daily basis when planning reading instruction for first graders who are developing as beginning readers? A. activities that introduce students to basic concepts about print B. activities that emphasize listening to and producing rhyming, alliteration, and similar forms of wordplay C. activities that promote students' development of decoding and other word analysis skills D. activities that emphasize memorization of lists of grade- level-appropriate sight words

C. Phonics and other word analysis skills, such as learning common inflectional endings and the orthographic rules governing their addition to words, are critical skills that promote beginning readers' development of proficiency in decoding. These skills in turn support students' development of reading fluency and comprehension. Daily, systematic, differentiated instruction and practice in these essential skills are necessary to ensure all students in a first-grade class develop into proficient readers. While some students at this level may still require differentiated or remedial instruction in concepts about print (A) and/or basic phonological awareness skills (B), these skills are generally mastered by most students in kindergarten, so they would not be taught to a whole first-grade class on a daily basis. Instruction in sight words is an important component of a research-based beginning reading program that is primarily phonics based; however, memorization of lists of sight words on a daily basis (D) is an ineffective instructional approach.

Frequent oral reading to kindergarten children using appropriate and expressive intonation and voices is likely to promote the students' reading development primarily by: A. improving their aural discrimination skills. B. explicitly teaching letter-sound correspondence. C. fostering their engagement in and love of reading. D. explicitly modeling phonological concepts such as word boundaries.

C. Promoting young children's motivation to engage in literacy-related tasks is an essential component of an effective kindergarten reading program. Research has shown that reading aloud to students promotes their vocabulary development as well as their interest in books and reading. Using expressive voices makes the activity even more engaging. Because the focus of a read-aloud is the construction of meaning, a read- aloud is not appropriate or effective to use for developing students' auditory discrimination skills (A), teaching alphabetic skills such as letter-sound correspondence (B), or teaching phonological skills such as detecting word boundaries (D).

In order to select a trade book that emphasizes predictability, a teacher should ensure that: A. the text includes some pictures or illustrations. B. the concepts in the text are at an appropriate level of difficulty for the target student(s). C. a phrase, rhyme, or sentence is repeated throughout the text. D. the length of the text is not likely to exceed the attention span of the target student(s).

C. Reading aloud predictable texts to pre-alphabetic and emergent readers is an effective way to promote young children's motivation to read and their development of concepts of print. The portion of the text that is predictable is generally a phrase, rhyme, or sentence that is repeated throughout the text, much like the refrain of a song. This repeated text is easy for young children to learn quickly, which allows them to "read" along with the teacher. A, B, and D all describe other important features common to books written for young children. However, these are not features that make the books predictable.

If a standardized test is said to lack reliability, the test: A. is not measuring what it is supposed to measure. B. has not proven to be useful as an instructional intervention. C. gives fluctuating scores in different administrations. D. has poor predictive value relative to students' classroom performance.

C. Reliability indicates the degree to which a test yields consistent results over successive administrations. If a test yielded fluctuating results, it would be considered to have low reliability. A relates to a test's content validity, while D relates to a test's predictive validity. Standardized tests are not intended to be used as interventions (B).

Which of the following strategies is likely to be most effective in promoting reluctant readers' interest in independent reading outside of school? A. Calculate numerical scores based on the number and difficulty level of the books students read at home and integrate the score into students' report card grade for reading. B. Encourage parents to give their children simple external rewards for at-home reading, such as an extra helping of a favorite treat. C. Encourage students and parents to read books together on a regular basis, either silently or aloud, and discuss their personal responses to each chapter or key event. D. Recommend that parents make their children's daily television-watching time contingent on their reading a specified number of pages first.

C. Reluctant readers are often reluctant to read independently at home because they have experienced some level of difficulty in reading. Encouraging students to read with a proficient reader, such as a parent or guardian, and to discuss key events in the text helps scaffold and support their comprehension of the text. Sharing personal responses to books with a partner also underscores that reading can be a highly social and enjoyable act; this in turn promotes positive dispositions toward reading. A and B are incorrect because providing extrinsic rewards to students for their reading may work as a short-term strategy with some students, but it is not effective in promoting a lifelong interest in or love of reading. Withholding a treat or reward until reading is complete (D) is another short-term strategy, but it may actually increase reluctant readers' negative feelings about reading instead of fostering their positive feelings.

A fifth-grade class silently reads an informational text. In subsequent informal assessments, several students are able to read the text orally with fluency but they demonstrate poor overall comprehension of the text. The teacher could most appropriately address these students' needs by adjusting future instruction in which of the following ways? A. using informational texts that are written at the students' independent reading level B. providing the students with explicit instruction in grade-level-appropriate test-taking strategies C. introducing a text's key vocabulary and guiding the students in close reading of key passages D. emphasizing reading skill-building activities that focus primarily on narrative texts

C. Since the students could read the text orally with fluency during subsequent informal assessments, the teacher needs to consider other factors that may have affected the students' comprehension of the text. Informational texts introduce vocabulary words that are likely to be unfamiliar to many students in a fifth- grade classroom. Informational texts may also include academic language structures with which students are unfamiliar. For students to be able to read and comprehend such a text independently, the teacher may need to model and provide students with practice in close reading of key passages of the text, as well as explicitly teach key vocabulary prior to reading. Using only informational texts that are written at students' independent reading levels (A) is not an effective option, as these texts are unlikely to include important grade-level vocabulary and thus may not match grade-level standards for that content area. Similarly, simply providing students with instruction in test-taking skills (B) or focusing on narrative texts for reading activities (D) would not help students develop the content-specific knowledge and vocabulary required at their grade level.

An English Language Learner reads academic texts fluently in her primary language but is struggling to understand her content-area textbooks in English. This student would likely benefit most from engaging in which of the following activities? A. translating textbook reading assignments from English into her primary language B. receiving reading comprehension instruction with texts written in her primary language C. learning to use metacognitive reading strategies with English text D. reading texts in her primary language that cover the same material as her English textbooks

C. The student reads academic texts well in her primary language, so she likely has both adequate background knowledge in the content areas and basic reading skills. The student would benefit from using metacognitive strategies such as self-monitoring to connect what she has already learned in the primary language with the content presented in the English textbooks. A, B, and D all focus on primary-language materials. While these strategies may be helpful in supporting the students' ongoing conceptual development with regard to specific content, they do not promote the students' ability to comprehend content-area textbooks in English.

A third-grade teacher has been conducting a series of ongoing assessments of a student's oral reading. Shown below is a sentence from a text, followed by a transcription of a typical example of the student's oral reading performance. Text: Her boots crunched through the snow. Student: Her boats crucked throw the snow. After reading the sentence, the student paused and then reread it without the teacher's prompting and self-corrected the errors. Based on this information, the teacher could best meet this student's needs by adjusting instruction in order to: A. enhance the student's oral vocabulary development. B. develop the student's ability to self- monitor comprehension. C. improve the student's decoding skills. D. promote the student's ability to track print.

C. The student's oral reading performance in this sample strongly suggests a lack of foundational knowledge in phonics and sight words. The miscues indicate serious decoding difficulties with various phonics elements, including lack of automaticity in decoding common vowel digraphs (reading boats for boots), common consonant digraphs (reading ck for ch), and complex consonant clusters (reading crucked for crunched). The student also misread a high-frequency sight word (through) that should have been mastered by the end of second grade. Conversely, this oral reading sample provides no evidence that the student has difficulty tracking print (D). And, by self-correcting his or her errors without prompting, the student demonstrates both understanding of the text's vocabulary (A) and the ability to self-monitor for comprehension (B).

A fifth-grade teacher plans to have students read a chapter about the American Revolutionary War from their social studies textbook. The following is an excerpt from the chapter. The Battle of Bunker Hill took place on June 17, 1775. At the time, the American army occupied the area from Cambridge to the Mystic River. American troops gathered in Cambridge Common on the evening of June 16, 1775, and set out for Bunker Hill. Upon reaching Bunker Hill, however, officers decided to move to Breed's Hill, a smaller hill closer to Boston. Based on this excerpt from the chapter, which of the following graphic organizers would best promote students' awareness of the chapter's text structure? A. outline B. Venn diagram C. timeline D. semantic map

C. The text structure of this passage is chronological, focusing on when troop movements took place leading up to the battle. Timelines are the most effective form of graphic organizer for conveying a chronology of events. Outlines (A) and semantic maps (D) are well suited to passages that are organized into different sections or categories of related content. Venn diagrams (B) visually display similarities and differences between two or more related topics.

Which of the following text features are students likely to find most useful when previewing informational texts such as library books for a research project? A. index B. bibliography C. glossary D. table of contents

D. A table of contents shows how the content of a text is organized and provides clues to the types of information the reader is likely to find in each section. Skimming a text's table of contents provides a reader with a quick and effective preview of the text's content. This allows a reader to determine whether the text might be useful for a particular research project. A is incorrect because a typical index lists all the topics covered by a text in great detail, but it is organized alphabetically rather than grouped in broad categories, so it is less effective as a tool for previewing texts. B and C are incorrect because bibliographies provide information about the sources the author(s) used in developing the text, while glossaries provide definitions of key terms used in the text.

Which of the following children is most in need of immediate intervention? A. a preschool child who has limited book-handling skills B. a kindergarten child who has limited ability to correlate alphabet letters with the sounds they make C. a first-grade student who still reads texts composed of single-syllable regular words and common sight words D. a second-grade student who still decodes words letter by letter

D. By second grade, students should be reading many regular words and sight words automatically. In addition, when reading new words that follow regular phonics patterns already taught, the students should be reading these patterns as chunks. A second-grade student who still decodes each word letter by letter is substantially behind grade-level benchmarks for oral reading fluency and requires immediate intervention. A preschool child who has limited book-handling skills (A), a kindergarten child who has not developed letter-sound correspondences for all letters (B), and a first-grade student who reads mostly decodable texts (C) are all performing within the average acceptable range for the respective grade levels.

A third-grade teacher observes that students who read aloud fluently also demonstrate greater comprehension of expository texts. The best explanation for this is that fluent readers: A. possess a self-awareness that allows them to use metacognitive skills efficiently. B. have already developed the base of background knowledge typically covered by textbooks. C. have well-developed skills for decoding any level of text word by word. D. are able to focus their full attention and cognitive resources on the meaning of a text.

D. Research has shown that fluent readers have higher levels of comprehension than readers who lack fluency. By the third grade, fluent readers have developed automaticity in decoding, which allows them to focus on the meaning of what they are reading rather than on expending all their effort and energy on decoding each individual word letter by letter. Achieving reading fluency does not necessarily mean that students know how to use metacognitive skills (A) or that they have already developed background knowledge at the level of a textbook (B). Having fluency also does not mean that a third-grade student is skilled enough to decode any level of text (C).

A first-grade teacher encourages beginning readers to "write" their own captions beneath their drawings. This practice is most likely to lead to which of the following? A. The students will tend to lose interest in writing because of their frustration with their lack of mastery of the English spelling system. B. The students' overall reading proficiency will be adversely affected by any spelling errors that go uncorrected. C. The students will tend to develop strong automatic word recognition skills from their interaction with print. D. The students' development of phonics knowledge will be reinforced as they experiment with their own phonetic spellings.

D. Research has shown that students' understanding of the alphabetic principle is enhanced and reinforced by having opportunities to apply their knowledge of letter-sound correspondences in both reading and writing—the latter by using phonetic spellings. A is incorrect because providing students with meaningful opportunities to write is motivating rather than discouraging for beginning readers. B is incorrect because first- grade teachers should not insist on the correct spelling of words that follow phonics patterns that have not yet been introduced. C is incorrect because students' experimental spellings would not consistently match the spellings used in conventional texts, so engaging in this writing activity would not promote automatic word recognition.

One of the most important purposes of a standardized Informal Reading Inventory (IRI) is: A. to establish how prior knowledge and text organization influence a student's reading comprehension. B. to determine how a student uses semantic, syntactic, and other text clues to deduce a word's meaning. C. to analyze how a student's silent reading comprehension is influenced by oral reading fluency. D. to establish a student's independent, instructional, and frustration reading levels.

D. Standardized Informal Reading Inventories (IRIs) are administered individually to students to establish the students' reading levels. The results of this type of assessment provide evidence to guide the selection of reading materials for students for instruction and/or interventions and to guide students in their selection of materials for independent reading. A is incorrect because the results of IRIs do not provide information about how a text's organization or a student's prior knowledge affect the student's comprehension. IRI results are also not helpful in determining what types of clues students use to determine the meaning of unfamiliar vocabulary (B) or in analyzing the relationship between a student's silent- and oral-reading proficiencies (C).

A fourth-grade English Language Learner is new to a school. Assessments suggest that the student can read orally with accuracy on grade level; however, the student's comprehension of grade-level textbooks fluctuates widely. Which of the following steps would be most appropriate for the teacher to take first in order to determine the cause of the student's difficulty? A. Assess the student's word analysis and decoding skills. B. Determine whether the student has a specific learning disability that affects language processing. C. Assess the student's level of first- language literacy. D. Determine whether the student has adequate vocabulary and background knowledge to support comprehension of the textbooks.

D. Vocabulary and background knowledge are critical components of reading comprehension, providing the foundation of the schema a student uses to construct meaning from the text. An English Language Learner's English vocabulary is likely to have gaps, particularly with respect to content-area topics that are not typical subjects of everyday conversations. An English Language Learner may be familiar with a content-area topic, but he or she may lack the relevant English vocabulary needed to comprehend an English- language text about that topic. In the scenario described, word analysis and decoding skills are not likely the cause of the student's difficulty (A), since the English Language Learner is able to read with grade-level accuracy. For the same reason, a learning disability that affects language processing (B) and the student's level of first- language literacy (C) are not likely factors impeding his or her reading comprehension.

Phonological awareness activities are: a) Oral b) Visual c) Both A and B d) Semantically based

a) Oral. Phonological awareness refers to an understanding of the sounds a word makes. While phonological awareness leads to fluent reading skills, activities designed to develop an awareness of word sounds are, by definition, oral.

Sight words are: a) Common words with irregular spelling b) Words that can easily be found on educational websites c) Any word that can be seen, including text words, words on signs, brochures, banners, and so forth d) There is no such thing; because oral language is learned before written language, all words are ultimately based on sound. The correct term is sound words and includes all words necessary to decode a particular text

a) Common words with irregular spelling. Sight words occur in many types of writing; they are high-frequency words. Sight words are also words with irregular spelling. Some examples of sight words include talk, some, and the. Fluent readers need to recognize these words visually.

Collaborative Strategic Reading (CSR) is a teaching technique that depends on two teaching practices. These practices are: a) Cooperative learning and reading comprehension b) Cooperative reading and metacognition c) Reading comprehension and metacognition d) Cooperative learning and metacognition

a) Cooperative learning and reading comprehension. Cooperative learning occurs when a group of students at various levels of reading ability have goals in common. Reading comprehension is achieved through reading both orally and silently, developing vocabulary, a reader's ability to predict what will occur in a piece of writing, a reader's ability to summarize the main points in a piece of writing, and a reader's ability to reflect on the text's meaning and connect that meaning to another text or personal experience.

It is the beginning of the school year. To determine which second-grade students might need support, the reading teacher wants to identify those who are reading below grade level. She works with students one at a time. She gives each child a book at a second-grade reading level and asks the child to read out loud for two minutes. Children who will need reading support are those who read: a) Fewer than 100 words in the time given b) Fewer than 200 words in the time given c) More than 75 words in the time given d) The entire book in the time given

a) Fewer than 100 words in the time given. At the beginning of the school year, second-grade students should be able to read 50-80 words per minute. By the time they are well into the school year, second-grade level reading is tracked at 85 words per minute.

Sea, and see, fair and fare, are called: a) Homophones b) Antonyms c) Homographs d) Twin words

a) Homophones. Homophones are a type of homonym that sound alike, but are spelled differently and have different meanings. Other examples are two, to, and too; their, they're, and there.

Americans have struggled with cigarettes far too long. Until now, it has been a personal choice to smoke (or not), but the time for change is rapidly approaching. Local legislation has already begun for schools, restaurants, arenas, and other public places to be smoke-free. Years ago cigarette smoking was presented by the media as being fashionable, even sexy. In magazines, movies, and later in television, celebrities would indulge themselves with a smoke and even be paid to endorse a brand. As recently as 1975, it was common for talk show hosts like Tom Snyder and Johnny Carson to keep a cigarette burning. Cigarette smoking in America has persisted in spite of frightening concerns like lung cancer and emphysema. Over the years, the tobacco industry has sought to diffuse strong evidence that smoking is harmful. However, the myth of "safe cigarettes," questions about nicotine addiction, and denials about the dangers of secondhand smoke have proven to be propaganda and lies. The thesis statement is: a) However, the myth of "safe cigarettes," questions about nicotine addiction, and denials about the dangers of secondhand smoke have proven to be propaganda and lies. b) Americans have struggled with cigarettes far too long c) Until now, it has been a personal choice to smoke (or not), but the time for change is rapidly approaching d) In magazines, movies, and later in television, celebrities would indulge themselves with a smoke and even be paid to endorse a brand

a) However, the myth of "safe cigarettes," questions about nicotine addiction, and denials about the dangers of secondhand smoke have proven to be propaganda and lies. A thesis statement offers a hypothesis or opinion that the remainder of the paper then sets out to prove. Oftentimes, the thesis statement also offers a clear road map of the paper, foreshadowing the focuses of the paragraphs that follow and the order in which they will appear.

All members of a group of kindergarten students early in the year are able to chant the alphabet. The teacher is now teaching the students what the alphabet looks like in written form. The teacher points to a letter and the students vocalize the correspondent sound. Alternatively, the teacher vocalizes a phoneme and a student points to it on the alphabet chart. The teacher is using ______________ in her instruction. a) Letter-sound correspondence b) Rote memorization c) Predictive analysis d) Segmentation

a) Letter-sound correspondence. Letter-sound correspondence relies on the relationship between a spoken sound or group of sounds and the letters conventionally used in English to write them.

Which of the following statements regarding the acquisition of language is false? a) Young children often have the ability to comprehend written language just as early as they can comprehend or reproduce oral language when given appropriate instruction b) Oral language typically develops before a child understands the relationship between spoken and written word c) Most young children are first exposed to written language when an adult reads aloud d) A child's ability to speak, read, and write depends on a variety of physiological factors, as well as environmental factors

a) Most adults can understand the relationship between oral and written language: components of oral language have representational symbols that can be written and decoded. However, most normally-developing children acquire spoken language first and begin to develop reading and writing skills as they approach school-age. Many children are first exposed to the concept of written language when an adult introduces books or other written texts. However, a child's ability to read and write develops over time and is dependent on the development of physiological processes such as hearing, sight, and fine motor skills for writing. Written language development also typically requires direct instruction. Most children must be taught to read and write and rarely learn these skills simply by observing others.

The kindergarten teacher is concerned about three of her students. While they are enthusiastic about writing, they do not always recognize letters, confusing b, d, and p, or e and o. They do, however, know which sounds go with certain letters when they are orally drilled. When they write, they appear to be attempting letter-sound associations. "Now I'm writing M," the teacher heard one boy say as he scripted a large N in the upper right corner of his paper. He studied it for a moment and added, "Nope, it needs another leg." The student then wrote an I beside the N. "There," he said, "Now you are an M. I can write the word 'man,' because now I have M." The child then moved to the lower left corner of the paper. "M-A-N," he said to himself, slowly pronouncing each sound. "I already have that M. Here is where the rest of the word goes." He turned the paper sideways and wrote N. The second child sang to herself as she gripped the crayon and scribbled lines here and there on her paper. Some of the lines resembled letters, but few actually were. Others were scribbles. As she "wrote," she seemed to be making up a story and seemed to believe she was writing the story down. The third child didn't vocalize at all while he worked. He gripped the paper and carefully wrote the same letter over and over and over and over. Sometimes the letter was large, sometimes tiny. He turned the paper in every direction so that sometimes the letter was sideways or upside down. Sometimes he flipped it backward. "What are you writing?" the teacher asked him. "My name," the child told her. The teacher then realized the letter was, indeed, the first letter of his name. She gently told him he had done a fine job of writing the first letter of his name. Did he want her to help him write the rest of it? "Nope," he cheerfully told her, "it's all here." He pointed at one of the letters and "read" his full name. He pointed at another letter and again seemed to believe it represented all the sounds of his name. The kindergarten teacher isn't certain if these children are exhibiting signs of a reading disability or other special needs. What should the teacher do? a) Nothing. These children are simply at an early stage in the reading/writing process b) Nothing. She doesn't want to have to tell the parents that their children are sub-par in terms of intelligence. They are perfectly nice children and can contribute to society in other ways. She resolves to give them extra attention in other areas to help them build confidence c) She should recommend that the parents take the children to be tested for a number of reading disorders, including dyslexia d) She should arrange a meeting between herself, the school psychologist, and the reading specialist to discuss the matter and resolve it using a three-pronged approach

a) Nothing. These children are simply at an early stage in the reading/writing process. When emergent readers become aware of the connections between letters and sounds, and between reading and writing, they want to practice the skills they see proficient readers use. While a proficient writer knows that letters are grouped into words and that words are constructed into sentences that move from left to right and from the top of the page to the bottom, an emergent reader/writer knows only that letters magically contain sounds that other people can read. It is necessary for children to pass through early stages in which they actually do write the initial letter of a word all over the page. Next, the emergent reader/writer will write the initial letter of many of the words that belong in the sentence and will write them sequentially. KJM, for example, might mean the cat chased a mouse.

A class is reading The Heart Is a Lonely Hunter. The teacher asks students to write a short paper explaining the story's resolution. She is asking them to locate and discuss the story's: a) Outcome b) Highest or most dramatic moment c) Plot d) Lowest point

a) Outcome. Story action can be analyzed in terms of rising action, story climax, falling action, and resolution. Rising action consists of those events that occur before and lead up to the story's most dramatic moment, or climax. The climax occurs toward the end of the book, but rarely, if ever, right at the end. Following the climax, the consequences of that dramatic moment are termed falling action. The story reaches resolution with the outcome of the falling action.

Phonemic awareness is a type of: a) Phonological awareness. Phonemic awareness is the ability to recognize sounds within words b) Phonics. It is a teaching technique whereby readers learn the relationship between letters and sounds c) Alphabetization. Unless a reader knows the alphabet, phonemic awareness is useless d) Syntactical awareness. Understanding the underlying structure of a sentence is key to understanding meaning

a) Phonological awareness. Phonemic awareness is the ability to recognize sounds within words. Segmenting words and blending sounds are components of phonemic awareness. Phonological awareness includes an understanding of multiple components of spoken language. Ability to hear individual words within a vocalized stream and ability to identify spoken syllables are types of phonological awareness.

Research indicates that developing oral language proficiency in emergent readers is important because: a) Proficiency with oral language enhances students' phonemic awareness and increases vocabulary b) The more verbally expressive emergent readers are, the more confident they become. Such students will embrace both Academic and Independent reading levels c) It encourages curiosity about others. With strong oral language skills, students begin to question the world around them. The more they ask, the richer their background knowledge d) It demonstrates to students that their ideas are important and worth sharing

a) Proficiency with oral language enhances students' phonemic awareness and increases vocabulary. Understanding that words are scripted with specific u representing specific sounds is essential to decoding a text. Students cannot effectively learn to read without the ability to decode. An enhanced vocabulary supports the act of reading; the larger an emergent reader's vocabulary, the more quickly he will learn to read. He will be able to decode more words, which he can organize into word families, which he can use to decode unfamiliar words.

An eighth-grade student is able to decode most words fluently and has a borderline/acceptable vocabulary, but his reading comprehension is quite low. He can be helped with instructional focus on: a) Strategies to increase comprehension and to build vocabulary b) Strategies to increase comprehension and to be able to identify correct syntactical usage c) Strategies to improve his understanding of both content and context d) Strategies to build vocabulary and to improve his understanding of both content and context

a) Strategies to increase comprehension and to build vocabulary. The student should receive instruction focused on just those areas in which he is exhibiting difficulty. Improved vocabulary will give him greater skill at comprehending the meaning of a particular text. Strategies focused on enhancing comprehension together with a stronger vocabulary will provide the greatest help.

A fourth-grade teacher is preparing her students for a reading test in which a number of words have been replaced with blanks. The test will be multiple-choice; there are three possible answers given for each blank. The teacher instructs the children to read all the possible answers and cross out any answer that obviously doesn't fit. Next, the students should "plug in" the remaining choices and eliminate any that are grammatically incorrect or illogical. Finally, the student should consider contextual clues in order to select the best answer. This is an example of: a) Strategy instruction b) Diagnostic instruction c) Skills instruction d) Multiple-choice instruction

a) Strategy instruction. Strategic instruction involves teaching a methodic approach to solving a reading problem. It consists of strategies done in steps which aid the reader in eliminating incorrect responses.

A reading teacher is working with a student who has just moved to Texas from Korea. The child knows very few words in English. The teacher offers her a picture book of Korean folk tales. Using words and gestures, the teacher asks her to "read" one folk tale. The child "reads" the familiar tale in Korean. The teacher then writes key English words on the board and asks the child to find those words in the book. When the child finds the words, they read them together. This strategy is: a) Useful. The child will feel more confident because the story is already familiar. She will also feel that the lesson is a conversation of sorts, and that she is communicating successfully. She will be motivated to learn the English words because they are meaningful and highly charged b) Useful. The teacher is learning as much as the child is. The teacher is learning about Korean culture and language, and she can apply this knowledge when teaching future Korean students c) Not very useful. The child needs to be exposed to as much American culture as possible. Encouraging her to remember her own culture will make her sad and will limit her curiosity about her new home d) Not very useful. The first things the child should learn are the letters of the alphabet and associative sounds. Only then can she begin to decipher an unfamiliar language

a) Useful. The child will feel more confident because the story is already familiar. She will also feel that the lesson is a conversation of sorts, and that she is communicating successfully. She will be motivated to learn the English words because they are meaningful and highly charged. As a newly arrived immigrant, the child feels overwhelmed. Presenting her with a book of folk tales from her country tells her that she needn't lose her culture in order to function in this one. It also comforts her by reminding her that the past and present are linked. Allowing her to speak in Korean helps her express herself without judgment or failure. Presenting her with an English vocabulary that is meaningful ensures that she will eagerly embrace these words, her first words in her new language.

The kindergarten teacher is concerned about three of her students. While they are enthusiastic about writing, they do not always recognize letters, confusing b, d, and p, or e and o. They do, however, know which sounds go with certain letters when they are orally drilled. When they write, they appear to be attempting letter-sound associations. "Now I'm writing M," the teacher heard one boy say as he scripted a large N in the upper right corner of his paper. He studied it for a moment and added, "Nope, it needs another leg." The student then wrote an I beside the N. "There," he said, "Now you are an M. I can write the word 'man,' because now I have M." The child then moved to the lower left corner of the paper. "M-A-N," he said to himself, slowly pronouncing each sound. "I already have that M. Here is where the rest of the word goes." He turned the paper sideways and wrote N. The second child sang to herself as she gripped the crayon and scribbled lines here and there on her paper. Some of the lines resembled letters, but few actually were. Others were scribbles. As she "wrote," she seemed to be making up a story and seemed to believe she was writing the story down. The third child didn't vocalize at all while he worked. He gripped the paper and carefully wrote the same letter over and over and over and over. Sometimes the letter was large, sometimes tiny. He turned the paper in every direction so that sometimes the letter was sideways or upside down. Sometimes he flipped it backward. "What are you writing?" the teacher asked him. "My name," the child told her. The teacher then realized the letter was, indeed, the first letter of his name. She gently told him he had done a fine job of writing the first letter of his name. Did he want her to help him write the rest of it? "Nope," he cheerfully told her, "it's all here." He pointed at one of the letters and "read" his full name. He pointed at another letter and again seemed to believe it represented all the sounds of his name. The teacher might best encourage the three students in the above example by: a) Suggesting they write an entire book rather than just a single page. This will build confidence, teach them sequencing, and encourage the young writers to delve deeper into their ideas. b) Ask the students to read their stories to her. Suggest they visit other children in the class and read to each of them. c) Contact the local newspaper and invite a reporter to visit her class and write a story about her emergent writers. In this way, they are sure to see themselves as "real writers" and will more fully apply themselves to the task. d) Invite all the parents to visit the class the following week. This will give all classmates, regardless of where they are on the learning spectrum, time to memorize their stories. This children will be very excited and will begin to see themselves as "real writers."

b) Ask the students to read their stories to her. Suggest they visit other children in the class and read to each of them. The teacher should encourage these students by "reading" what they have written, even if what she reads is incorrect. She might misread KJM as Kathy jumped rope with Mandy. Most children will not be upset by this, but will connect the teacher's misreading by reading what the letters really mean.

TRain, BRain, SPring. The capitalized letters are examples of: a) Consonant digraph b) Consonant blend c) Consonant shift d) Continental shift

b) Consonant blend. Consonant blend refers to a group of consonants in which each letter represent a separate sound

The students in the above question are most likely preparing to write a(n) ___________ essay: a) Personal b) Expository c) Literary d) Narrative

b) Expository. Expository essays clarify an idea, explain, or interpret facts. The position the author takes is often supported with statistics, quotations, or other evidence researched from a variety of sources.

Americans have struggled with cigarettes far too long. Until now, it has been a personal choice to smoke (or not), but the time for change is rapidly approaching. Local legislation has already begun for schools, restaurants, arenas, and other public places to be smoke-free. Years ago cigarette smoking was presented by the media as being fashionable, even sexy. In magazines, movies, and later in television, celebrities would indulge themselves with a smoke and even be paid to endorse a brand. As recently as 1975, it was common for talk show hosts like Tom Snyder and Johnny Carson to keep a cigarette burning. Cigarette smoking in America has persisted in spite of frightening concerns like lung cancer and emphysema. Over the years, the tobacco industry has sought to diffuse strong evidence that smoking is harmful. However, the myth of "safe cigarettes," questions about nicotine addiction, and denials about the dangers of secondhand smoke have proven to be propaganda and lies. This is a(n) ________________ essay: a) Compare/contrast b) Persuasive c) Narrative d) Analytic

b) Persuasive. A persuasive essay takes a strong position about a controversial topic and offers factual evidence to support this position. The goal of a persuasive paper is to convince the audience that the claim is true based on the evidence provided.

Which of the following best explains the importance prior knowledge brings to the act of reading? a) Prior knowledge is information the student gets through researching a topic prior to reading the text. A student who is well-prepared through such research is better able to decode a text and retain its meaning. b) Prior knowledge is knowledge the student brings from previous life or learning experiences to the act of reading. It is not possible for a student to fully comprehend new knowledge without first integrating it with prior knowledge. c) Prior knowledge is predictive. It motivates the student to look for contextual clues in the reading and predict what is likely to happen next. d) Prior knowledge is not important to any degree to the act of reading, because every text is self-contained and therefore seamless. Prior knowledge is irrelevant in this application.

b) Prior knowledge is knowledge the student brings from previous life or learning experiences to the act of reading. It is not possible for a student to fully comprehend new knowledge without first integrating it with prior knowledge. Prior knowledge, which rises from experience and previous learning, provides a framework by which new knowledge gained from the act of reading can be integrated. Every act of reading enriches a student's well of prior knowledge and increases that student's future ability to comprehend more fully any new knowledge acquired through reading.

A fifth grader has prepared a report on reptiles, which is something he knows a great deal about. He rereads his report and decides to make a number of changes. He moves a sentence from the top to the last paragraph. He crosses out several words and replaces them with more specific words. He circles key information and draws and arrow to show another place the information could logically be placed. He is engaged in: a) Editing b) Revising c) First editing, the revising d) Reviewing

b) Revising. Revision (literally, re+vision) is the act of "seeing again." When revising, writers examine what they have written in order to improve the meaning of the work. Fine-tuning word choices, moving information to another location, and adding or deleting words are all acts of revision.

Which is greater, the number of English phonemes or the number of letters in the alphabet? a) The number of letters in the alphabet, because they can be combined to create phonemes b) The number of phonemes. A phoneme is the smallest measure of language sound c) They are identical; each letter "owns" a correspondent sound d) Neither. Phonemes and alphabet letters are completely unrelated

b) The number of phonemes. A phoneme is the smallest measure of language sound. English language phonemes, about 40 in number, are composed of individual letters as well as letter combinations. A number of letters have more than one associated sound. For example, "c" can be pronounced as a hard "c" (cake) or a soft "c" (Cynthia). Vowels in particular have a number of possible pronunciations.

"Language load" refers to: a) The basic vocabulary words a first grader has committed to memory b) The number of unrecognizable words an English Language Learner encounters when reading a passage or listening to a teacher c) The damage that carrying a pile of heavy books could cause to a child's physique d) The number of different languages a person has mastered

b) The number of unrecognizable words an English Language Learner encounters when reading a passage or listening to a teacher. Language load is one of the barriers English Language Learners face. To lighten this load, a teacher can rephrase, eliminate unnecessary words, divide complex sentences into smaller units, and teach essential vocabulary before the student begins the lesson.

Reading comprehension and vocabulary can best be assessed: a) With brief interviews and tests every two months to determine how much learning has taken place. Students learn in spurts, and in-depth assessments of comprehension and vocabulary are a waste of time b) Through a combination of standardized testing, informal teacher observations, attention to grades, objective-linked assessments, and systematized charting of data over time c)By giving students weekly self-assessment rubrics to keep them constantly aware of and invested in their own progress d) By having students retell a story or summarize the content of an informational piece of writing. The degree to which the material was comprehended, and the richness or paucity of vocabulary used in such work, provides efficient and thorough assessment

b) Through a combination of standardized testing, informal teacher observations, attention to grades, objective-linked assessments, and systematized charting of data over time. Reading comprehension and vocabulary cannot be sufficiently assessed with occasional, brief studies. Continuous observation, high-stakes and standardized testing, attention to grades, and closely tracking the outcomes of objective-linked assessments are interrelated tools that, when systemically organized, offer a thorough understanding of students' strengths and weaknesses

The purpose of corrective feedback is: a) To provide students with method for explaining to the teacher or classmates what a passage was about b) To correct an error in reading a student has made, specifically clarifying where and how the error was made so that the student can avoid similar errors in the future c) To provide a mental framework that will help the student correctly organize new information d) To remind students that error is essential in order to truly understand and that it is not something to be ashamed of

b) To correct an error in reading a student has made, specifically clarifying where and how the error was made so that they student can avoid similar errors in the future. A reading teacher offers corrective feedback to a student in order to explain why a particular error in reading is, in fact, an error. Corrective feedback is specific; it located where and how the student went astray so that similar errors can be avoided in future reading.

A cloze test evaluates a student's: a) Reading fluency b) Understanding of context and vocabulary c) Phonemic skills d) Ability to apply the alphabetic principle to previously unknown materials

b) Understanding of context and vocabulary. In a cloze test, a reader is given a text with certain words blocked out. The reader must be able to determine probable missing words based on contextual clues. In order to supply these words, the readers must already know them.

Context clues are useful in: a) Predicting future action b) Understanding the meaning of words that are not familiar c) Understanding character motivation d) Reflecting on a text's theme

b) Understanding the meaning of words that are not familiar. Context cues offer insight into the probably meaning of unfamiliar words.

"Decoding" is also called: a) Remediation b) Deciphering c) Alphabetic principle d) Deconstruction

c) Alphabetic principle. The act of decoding involved first recognizing the sounds individual letters and letter groups make, and then blending the sounds to read the word. A child decoding the word spin, for example, would first pronounce sp/i/n as individual sound units. She them would repeat the sounds, smoothly blending them. Because decoding involves the understanding letters and their sounds, it is sometimes known as the alphabetic principle.

Caret, carrot, to, two, and too share something in common. They: a) Are nouns b) Are monosyllabic c) Are homophones d) Represent things in nature

c) Are homophones. Homophones are words that are pronounced the same, but differ in meaning. For example, a bride wears a 2 caret ring, but eats a carrot.

Another name for a persuasive essay is: a) Dynamic essay b) Convincing essay c) Argumentative essay d) Position paper

c) Argumentative essay. The goal of a persuasive essay is to convince the reader that the author's position or opinion on a controversial topic is correct. That opinion or position is called the argument. A persuasive essay argues a series of points, supported by facts and evidence.

A class will visit an assisted living facility to interview residents about their lives. Each group of three has selected a theme such as love, work, or personal accomplishment and written several questions around the theme. Next each group practices interviewing one another. The teacher then asks all the students to discuss the questions that caused them to respond most thoughtfully, as well as those they were less inspired by. The students decided the questions that were easiest to respond to asked for very specific information; for example, one inspiring question was, "Please tell me about something you learned to do as a child that affected the direction of your life." Those that were uninspiring were too broad, for example, "Please tell me about your happiest memory." The genre the teacher expects is: a) Memoir b) Historical fiction c) Biography d) Autobiography

c) Biography. A biography relates information about part of the life of an individual. An autobiography is a biography about the writer's own life. A memoir is also autobiographical, but focuses on a theme. Historical fiction uses a setting or event based in historical fact as the background for characters and/or action that is invented.

Since, whether, and accordingly are examples of which type of signal words? a) Common, or basic, signal words b) Compare/constrast words c) Cause-effect words d) Temporal sequencing words

c) Cause-effect words. Signal words give the reader hints about the purpose of a particular passage. Some signal words are concerned with comparing/contrasting, some with cause and effect, some with temporal sequencing, some with physical location, and some with a problem and its solution. The words since, whether, and accordingly are words used when describing an outcome. Outcomes have causes.

Dr. Jenks is working with a group of high school students. They are about to read a science book about fossils. Before they begin, she writes the words stromatolites, fossiliferous, and eocene on the board. She explains the meaning of each word. These words are examples of: a) Academic words b) Alliteration c) Content-specific words d) Ionization

c) Content-specific words. Because these words are specific to paleontology, it's unlikely the students know their meanings. Without understanding what these words mean, the students would not be able to understand the content of the passage they were about to read.

A teacher is working with a student who is struggling with reading. The teacher gives him a story with key words missing: The boy wanted to take the dog for a walk. The boy opened the door. The _________ ran out. The ________ looked for the dog. When he found the dog, he was very __________. The student is able to fill in the blanks by considering: a) Syntax. Oftentimes, word order gives enough clues that a reader can predict what happens next. b) Pretext. By previewing the story, the student can deduce the missing words. c) Content. By considering the other words in the story, the student can determine the missing words. d) Sequencing. By putting the ideas in logical order, the student can determine the missing words

c) Context. By considering the other words in the story, the student can determine the missing words. The student is depending on the information supplied by the rest of the story. This information puts the story into context.

The teacher and her students brainstorm a list of talents, skills, and specialized knowledge belonging to members of the class. Some of the items on the list include how to make a souffle, how to juggle, and how to teach a dog to do tricks. One student knows a great deal about spiders, and another about motorcycles. She asks each student to write an essay about something he or she is good at or knows a great deal about. What kind of essay is she asking the students to produce? a) Cause and effect b) Compare/contrast c) Example d) Argumentative

c) Example. Example essays, also called illustration essays, are simple, straightforward pieces that depend on clearly described examples to make their points. An example essay isn't trying to convince the reader (argumentative), compare similar or dissimilar things (compare/contrast), or point to relationships such as cause and effect. Often, example essays teach the reader how to accomplish something or about something.

Using brain imaging, researchers have discovered that dyslexic readers use the ______________ side(s) of their brains, while non-dyslexic readers use the ______________ side(s) of their brains. a) Left; right b) Right; left c) Right and left; left d) Right; left and right

c) Right and left; left. Researchers have discovered through brain imaging that a dyslexic reader uses both sides of the brain. Non-dyslexic readers use only the left side.

A third-grade teacher has several students reading above grade level. Most of the remaining students are reading at grade level. There are also a few students reading below grade level. She decides to experiment. Her hypothesis is that by giving the entire class a chapter book above grade level, high-level readers will be satisfied, grade-level readers will be challenged in a positive way, and students reading below grade level will be inspired to improve. Her method is most likely to: a) Succeed, producing students reading at an Instructional reading level. High-level readers will be happy to be given material appropriate to their reading level. Grade-level readers will challenge themselves to improve reading strategies in order to master the text. Because only a few of the students are reading below grade level, the other students, who feel happy and energized, will inspire the slower readers by modeling success. b) Succeed, producing students reading at an Independent reading level. High-level readers will independently help grade-level readers who will, in turn, independently help those below grade level. c) Fail, producing students at a Frustration reading level. Those reading below grade level are likely to give up entirely. Those reading at grade level are likely to get frustrated and form habits that will actually slow down their development. d) Fail, producing students reading at a Chaotic reading level. By nature, children are highly competitive. The teacher has not taken into consideration multiple learning styles. The children who are at grade level will either become bitter and angry at those whose reading level is above grade level or simply give up. The children reading below grade level will not be able to keep up and will in all likelihood act out their frustration or completely shut down.

c) Fail, producing students at a Frustration reading level. Those reading below grade level are likely to give up entirely. Those reading at grade level are likely to get frustrated and form habits that will actually slow down their development. Giving students texts that are too far beyond their reach produces frustrated readers. In an effort to succeed, frustrated writers are likely to apply strategies that have worked for them in the past but cannot work in this case because the test is simply beyond them. Looking for contextual clues to understand the meaning of unfamiliar words requires that most of the words in the passage are familiar. Breaking unfamiliar words into individual phonemes or syllables can be effective, but not if the number of such words is excessive. In this case, students below reading level and students at reading level will become frustrated when the skills that have worked for them in the past now fail.

A teacher is teaching students analogizing. She is teaching them to: a) Identify and use metaphors b) Identify and use similes c) Identify and use groups of letters that occur in a word family d) Identify and use figures of speech

c) Identity and use groups of letters that occur in a word family. Analogizing is based on recognizing the pattern of letters in words that share sound similarities. If the pattern is found at the end of a family of words, it is called a rhyme. Some examples of rhyme are rent, sent, bent, and dent. If the pattern is found at the beginning of the family of words, it is frequently a consonant blend such as street, stripe, or strong, in which all the letters are pronounced, or the pattern is a consonant digraph, in which letters are taken together to represent a single sound such as in phone, phonics, and phantom.

A student encounters a multisyllabic word. She's not sure if she's seen it before. What should she do first? What should she do next? a) Locate familiar word parts, then locate the consonants b) Locate the consonants, then locate the vowels c) Locate the vowels, the locate familiar word parts d) Look it up in the dictionary, the write down the meaning

c) Locate the vowels, then locate familiar word parts. Syllables are organized around vowels. In order to determine the syllables, this student should begin by locating the vowels. It's possible to have a syllable that is a single vowel (a/gain). It isn't possible to have a syllable that is a single consonant. Once the word has been broken into its component syllables the reader is able to study the syllables to find ones that are familiar and might give her a clue as to the word's meaning, such as certain prefixes or suffixes.

A teacher has a child who does not volunteer in class. When the teacher asks the student a question the student can answer, she does so with as few words as possible. The teacher isn't sure how to best help the child. She should: a) Leave the child alone. She is clearly very shy and will be embarrassed by having attention drawn to her. She is learning in her own way. b) Ask two or three highly social children to include this girl in their activities. She is shy, and she probably won't approach them on her own. c) Observe the child over the course of a week or two. Draw her into conversation and determine if her vocabulary is limited, if she displays emotional problems, or if her reticence could have another cause. Note how the child interacts with others in the class. Does she ever initiate conversation? If another child initiates, does she respond? d) Refer her to the school counselor immediately. It is clear the child is suffering from either a low IQ or serious problems at home.

c) Observe the child over the course of week or two. Draw her into conversation and determine if her vocabulary is limited, if she displays emotional problems, or if her reticence could have another cause. Note how the child interacts with others in the class. Does she ever initiate conversation? If another child initiates, does she respond? Until the teacher monitors the child's verbal abilities and habits, she cannot determine if the lack of interaction suggests a learning disability, an emotional problem, or simply a shy personality. The teacher should informally observe the child over a period of time, noting if and when she initiates or responds to oral language, if she is reading with apparent comprehension, if her vocabulary is limited, and the degree to which the child is interested in understanding.

A high school class reads an essay about the possible effects of sexual activity on teens. The author's position is very clear: She believes young people should avoid sex because they aren't mature enough to take the necessary steps to remain safe. The author cites facts, research studies, and statistics to strengthen her position. This type of writing is called: a) Expository b) Narrative c) Persuasive d) Didactic

c) Persuasive. The author is hoping to persuade or convince young readers to avoid sex by providing them with facts as well as by using rhetorical devices such as dispelling opposing arguments.

In preparation for writing a paper, a high school class has been instructed to skim a number of Internet and print documents. They are being asked to: a) Read the documents several times, skimming to a deeper level of understanding each time b) Read the documents quickly, looking for those that offer the most basic, general information c) Read the documents quickly, looking for key words in order to gather the basic premise of each d) Read the documents carefully, looking for those that offer the most in-depth information

c) Read the documents quickly, looking for key words in order to gather the basic premise of each. Skimming allows a reader to quickly gain a broad understanding of a piece of writing in order to determine if a more thorough reading is warranted. Skimming allows students who are researching a topic on the Internet or in print to consider a substantial body of information in order to select only that of a particular relevance.

At the beginning of each month, Mr. Yi had Jade read a page or two from a book she hasn't seen before. He notes the total number of words in the section, and also notes the number of times she leaves out or misreads a word. If Jade reads the passage with less than 3% error, Mr. Yi is satisfied that Jade is: a) Reading with full comprehension b) Probably bored and should try a more difficult book c) Reading at her Independent reading level d) Comfortable with the syntactical meaning

c) Reading at her independent reading level. When reading independently, students are at the correct level if they read with at least 97% accuracy.

A reading teacher is assessing an eighth grader to determine her reading level. Timed at a minute, the student reads with 93% accuracy. She misreads an average of seven words out of 100. What is her reading level? a) She is reading at a Frustration level b) She is reading at an Excellence level c) She is reading at an Instructional level d) She is reading at an Independent level

c) She is reading at an Instructional level. In one minute, a student who misreads one or less than one words per twenty words, or with 95%-100% accuracy, is at an Independent reading level. A student who misreads one or less than one word per ten words, or with 90%-95% accuracy, is at an Instructional level. A student misreading more than one word out of ten, or with less than 90% accuracy, is at a Frustration level.

Which text(s) are likely to foster the greatest enthusiasm for reading and literature among students? a) Free choice of reading texts, provided that students complete class assignments, projects, and discussions b) An all-in-one textbook that includes all reading materials for the year, study guides, and sample test questions c) A variety of texts, including books, magazines, newspapers, stories from oral traditions, poetry, music, and films d) A small selection of current best-selling books for children, some of which the children may already have read and liked

c) Students can easily become bored or disinterested in reading if they are not exposed to a variety of reading texts. Also, reading can be overwhelming or frustrating for students who are still learning to read fluently or to comprehend what they read. By incorporating media, oral stories, and various types of print, students of all ability levels can build both fluency and comprehension skills. This approach also enables the teacher and students to discuss the relationship between all aspects of literacy, including speaking, listening, thinking, viewing, and reading.

A class will visit an assisted living facility to interview residents about their lives. Each group of three has selected a theme such as love, work, or personal accomplishment and written several questions around the theme. Next each group practices interviewing one another. The teacher then asks all the students to discuss the questions that caused them to respond most thoughtfully, as well as those they were less inspired by. The students decided the questions that were easiest to respond to asked for very specific information; for example, one inspiring question was, "Please tell me about something you learned to do as a child that affected the direction of your life." Those that were uninspiring were too broad, for example, "Please tell me about your happiest memory." The teach wants the students to apply what they've learned across content areas. Which of the following strategies would be most effective? a) Students will interview a family member, asking the same questions b) Students will write a personal piece in which they address the same questions c) Students will do online research about the cultural, economic, or political events that were occurring during the specific time about which they've written d) Students pretend to be interviewee and rewrite the piece from a first person point of view

c) Students will do online research about the cultural, economic, or political events that were occurring during the specific time about which they've written. By researching the historic setting that cradled the events their interviewee discussed, students are simultaneously broadening their understanding of the context and working in a different content area.

For their monthly project, a group of students can choose to read and respond to one book on a list supplied by their teacher. The books are grouped according to genre. Most students choose books listed under the genre that is described as "modern-day stories that are not true, but seem as though they could really happen." Which genre did most of the students choose from? a) Historical fiction b) Autobiography c) Realistic fiction d) Fantasy

c) There are many genres from which students can choose to read. The most elemental distinction between genres consists of fiction and non-fiction, the latter referring to stories or texts that are true, or factual. Fictional texts can fall into a variety of categories. Realistic fiction seems as though it could be true. These stories involve realistic characters and settings with which readers can often identify. This type of fiction can treat different subjects, but it still must be relatable in nature.

A reading teacher feels that some of his strategies aren't effective. he has asked a specialist to observe him and make suggestions as to how he can improve. The reading specialist should suggest that first: a) The teacher set up a video camera and record several sessions with different students for the specialist to review. The presence of an observer changes the outcome; if the specialist is in the room, it will negatively affect the students' ability to read b) The teacher reflects on his strategies himself. Which seem to work? Which don't? Can the teacher figure out why? It's always best to encourage teachers to find their own solutions so that they can handle future issues themselves c) They meet to discuss areas the teacher is most concerned about and decide on the teacher's goals d) The specialist should arrive unannounced to observe the teacher interacting with students. This will prevent the teacher from unconsciously over-preparing

c) They meet to discuss areas the teacher is most concerned about and decide on the teacher's goals. In order to best achieve goals, those goals must be understood and established.

Of the three tiers of words, the most important words for direct instruction are: a) Tier-one words b) Common words c) Tier-two words d) Words with Latin roots

c) Tier-two words. Tier-two words are words that are used with high frequency across a variety of disciplines or words with multiple meanings. They are characteristic of mature language users. Knowing these words is crucial to attaining an acceptable level of reading comprehension and communication skills.

The following is/are (an) element(s) of metacognition: a) A reader's awareness of herself as a learner b) A reader's understanding of a variety of reading strategies and how to apply them to comprehend a text c) A reader who is conscious about remembering what has been read d) All of the above

d) All of the above. Metacognition means a reader's awareness of her own reading processes as she improves reading comprehension. Other elements of metacognition include awareness of areas in the text where the reader fails to comprehend and an understanding of how the text is structured.

A kindergarten teacher pronounces a series of word pairs for her students. The students repeat the pairs. Some of the of the pairs rhyme (see/bee) and some of the pairs share initial sounds but do not rhyme (sit, sun). The students help her separate the word pairs into pairs that rhyme and pairs that do not. Once the students are able to distinguish between two words that rhyme and two words that do not, the teacher says a word and asks them to provide a rhyme. When she says cat a child responds with fat. When she says sing a child offers thing. How does this strictly oral activity contribute to the children's ability to read? a) It doesn't. Oral activities must have a written component to be useful to emergent readers b) It is helpful in that it demonstrates how different sounds are made with different letters c) It actually discourages children from reading. By emphasizing orality over literacy, the teacher is suggesting to the children that reading is not an important skill d) Being able identify rhyme is an important element of phonological awareness

d) Being able identify rhyme is an important element of phonological awareness. Young children use language in a solely oral way. Oral language is composed of separate sounds that are represented in written form by the alphabet. In order to read, a child must first have a sense of the sounds that are used in English (phonological awareness). By helping children hear the difference between rhyming and non-rhyming words, the teacher is preparing them to make the transition to sound-letter association and word families

PHone, THey, CHurch. The capitalized letters in these words are examples of: a) Consonant blend b) Consonant shift c) Continental shift d) Consonant digraph

d) Consonant digraph. A consonant digraph is a group of consonants in which all letters represent a single sound.

A reading teacher is working with a student who has just moved to Texas from Korea. The child knows very few words in English. The teacher offers her a picture book of Korean folk tales. Using words and gestures, the teacher asks her to "read" one folk tale. The child "reads" the familiar tale in Korean. The teacher then writes key English words on the board and asks the child to find those words in the book. When the child finds the words, they read them together. The teacher was using what kind of load? a) Language load b) Cognitive load c) Bilingual load d) Cultural load

d) Cultural load. Cultural load is concerned with how the relationship between language and culture can help or hinder learning. By using the Korean folk tale, the teacher offered the child the opportunity to learn new words in a context that was culturally familiar. By demonstrating respect for her student's culture, she helped lighten the cultural load.

When should students learn how to decode? a) Decoding is the most basic and essential strategy to becoming a successful readers. It should be introduced to kindergartners during the first two weeks of school b) Decoding is not a teachable skill. It is an unconscious act and is natural to all learners c) Decoding should be taught only after children have mastered every letter-sound relationship as well as every consonant digraph and consonant blend. They should be able to recognize and say the 40 phonemes common to English words and be able to recognize at least a dozen of the most common sight words d) Decoding depends on an understanding of letter-sound relationships. As soon as a child understands enough letters and their correspondent sounds to read a few words, decoding should be introduced

d) Decoding depends on an understanding of letter-sound relationships. As soon as a child understands enough letters and their correspondent sounds to read a few words, decoding should be introduced. The act of decoding involves first recognizing the sounds individual letters and letter groups in a word make and then blending the sounds to read the word. It's important to introduce the strategy as soon as a child knows enough letters and their corresponding sounds to read simple words.

Examples of CVC words include: a) Add, pad, mad b) Cat, tack, act c) Elephant, piano, examine d) Dog, sit, leg

d) Dog, sit, leg. CVC words are composed of a consonant, a vowel, and a consonant. To learn to read them. students must be familiar with the letters used and their sounds. A teacher can present a word like sit to students who also know the consonants b/f/h/p and ask them to create a word family of other CVC words. The students will be able to read bit, fit, hit, and pit because they are similar to the word sit they have just learned.

Word-recognition ability is: a) Equally important to all readers b) Used only by fluent readers c) Another term for "word attack" d) Especially important to English Language Learners and students with reading disabilities

d) Especially important to English Language Learners and students with reading disabilities. Word recognition is required for reading fluency and is important to all readers, but it is especially so to English Language Learners and students with reading disabilities. It can be effectively taught through precisely calibrated word study instruction designed to provide readers with reading and writing strategies for successful word analysis.

The term "common words" means: a) One-syllable words with fewer than three letters. Some examples are it, an, a, I, go, to, and in. They are the first words an emergent writer learners b) One-syllable words with fewer than five letters. Some examples include sing, goes, sit, rock, walk, and took c) Words that are ordinary or unexceptional; because they tend to flatten a piece of writing, they should be avoided d) Familiar, frequently used words that do not need to be taught beyond primary grades

d) Familiar, frequently used words that do not need to be taught beyond primary grades. Common or basic words are the first tier of three-tier words. These words are widely used across the spoken and written spectrum. Some examples are walk, go, wish, the, look, happy, and always. This essential vocabulary is taught early in a reader's instruction, and beyond that it need not be taught.

Which of the following choices will be most important when designing a reading activity or lesson for students? a) Selecting a text b) Determining the number of students participating c) Analyzing the point in the school year at which the lesson is given d) Determining a purpose for instruction

d) It is impossible to include every text desired into the language curriculum - there are simply too many good books, stories, poems, speeches, and media available. Teachers must first think about what skills their students need to acquire. as well as what skills they have already mastered. In designing activities for class, a good teacher will start first with the purpose for instruction (or perceiving oral or visual text such as video or music). For example, purposes of reading can include: reading for information; reading for enjoyment; understanding a message; identifying main or supporting ideas; or developing an appreciation for artistic expression/perception. Once the purpose or intended learning outcome has been identified, the teacher will have a much better idea of which texts, strategies, and activities will support that purpose.

The most effective strategy for decoding sight words is: a) Segmenting sight words into syllables. Beginning readers are understandably nervous when encountering a long word that isn't familiar. Blocking off all but a single syllable at a time renders a word manageable and allows the reader a sense of control over the act of reading b) Word families. By grouping the sight word with similar words, patterns emerge c) A phonemic approach. When students understand the connection between individual words and their sounds, they will be able to sound out any sight word they encounter d) None; sight words cannot be decoded. Readers must learn to recognize these words as wholes on sight

d) None; sight words cannot be decoded. Readers must learn to recognize these words as wholes on sight. Sight words have irregular spelling. Segmenting them into syllables or using a phonemic approach are ineffective strategies to aid a reader in recognizing a sight word, because these approaches depend on rules a sight word doesn't follow. Word families group words that share common patterns of consonants and vowels. The spelling of those words is therefore regular, because they follow a predictable pattern. Sight words are irregular and do not follow a predictable pattern and must be instantaneously recognized for writing fluency. No decoding is useful.

Bi, re, and un are: a) Suffixes, appearing at the beginning of base words to change their meaning b) Suffixes, appearing at the end of base words to enhance their meaning c) Prefixes, appearing at the beginning of base words to emphasize their meaning d) Prefixes, appearing at the beginning of base words words to change their meanings

d) Prefixes, appearing at the beginning of base words to change their meanings. Suffixes appear at the end of words. Prefixes are attached to the beginning of words to change their meanings. Un+happy, bi+monthly, and re+examine are prefixes that, by definition, change the meaning of the words to which they are attached.

A teacher is working with a group of third graders at the same reading level. Her goal is to improve reading fluency. She asks each child in turn to read a page from a book about mammal young. She asks the children to read with expression. She also reminds them they don't need to stop between each word; they should read as quickly as they comfortably can. She cautions them, however, not to read so quickly that they leave out or misread a word. The teacher knows the components of reading fluency are: a) Speed, drama, and comprehension b) Cohesion, rate, and prosody c) Understanding, rate, and prosody d) Rate, accuracy, and prosody

d) Rate, accuracy, and prosody. Fluent readers are able to read smoothly and comfortably at a steady pace (rate). The more quickly a child reads, the greater the chance of leaving out a word or substituting one word for another (for example, sink instead of shrink). Fluent readers are able to maintain accuracy without sacrificing rate. Fluent readers also stress important words in a text, group words into rhythmic phrases, and read with intonation (prosody).

Round-robin reading refers to the practice of allowing children to take turns reading portions of a text aloud to the rest of the group during class. Which of the following statements is least true about this practice? a) Students have the chance to practice reading aloud with this strategy b) This practice is ineffective in its use of time, leaving students who are not reading aloud to become bored or daydream c) Round-robin reading lacks the creativity or engaging qualities that will interest students in building literacy skills d) This practice helps students feel comfortable with reading aloud due to continuous practice and encouragement from the teacher and peers

d) Round-robin reading is a common practice in language arts classes and has been for many years. In this process, students take turns reading aloud for their peers. Other students are asked to follow along silently in their texts while a peer is reading. This strategy does provide a way for students to read texts in class and include as many students as possible, which is often the intended outcome. However, this process often creates a boring atmosphere, since only one student at a time is actively engaged. While that student is reading, other students may become distracted by their own thoughts, other school work, or off-task interaction with each another; all of these issues subvert the intended outcome of the process. There is rarely enough time for each student to practice reading aloud to build students' reading fluency or comprehension in significant ways.

Americans have struggled with cigarettes far too long. Until now, it has been a personal choice to smoke (or not), but the time for change is rapidly approaching. Local legislation has already begun for schools, restaurants, arenas, and other public places to be smoke-free. Years ago cigarette smoking was presented by the media as being fashionable, even sexy. In magazines, movies, and later in television, celebrities would indulge themselves with a smoke and even be paid to endorse a brand. As recently as 1975, it was common for talk show hosts like Tom Snyder and Johnny Carson to keep a cigarette burning. Cigarette smoking in America has persisted in spite of frightening concerns like lung cancer and emphysema. Over the years, the tobacco industry has sought to diffuse strong evidence that smoking is harmful. However, the myth of "safe cigarettes," questions about nicotine addiction, and denials about the dangers of secondhand smoke have proven to be propaganda and lies. The next three paragraphs in the essay will most likely address: a) Smoking as a personal choice, changes in local legislation, and how fashionable smoking once was b) How fashionable smoking once was, talk show hosts smoking on air, the myth of "safe cigarettes" c) Propaganda and lies, the myth of "safe cigarettes," and how long Americans have struggled with cigarettes d) The myth of "safe cigarettes," questions about nicotine addiction, and the dangers of secondhand smoke

d) The myth of "safe cigarettes," questions about nicotine addiction, and the dangers of secondhand smoke. These three foci are presented in the thesis statement in this order and will be fleshed out in the following three paragraphs as the body of the essay.


Kaugnay na mga set ng pag-aaral

International Business Chapter 18

View Set

The Progressive Era - Progressivism Under Taft

View Set

Choosing Language for Context and Purpose

View Set

Urinary System Anatomy and Physiology II

View Set

Adult Health Chapter 41 EAQ Questions, adult sh*t

View Set

Honors Algebra 2/ Trigonometry (Justice) Chapter 3 and 4 Notes

View Set

BIO 264 Module 12 Special Senses Hearing & Equilibrium

View Set

Unit Test for The first half of the Twentieth Century

View Set

GL19 U7 (PowerPoint) CH04 Concepts Exam

View Set

Databases - Adaptive Reading Assignment

View Set

7.2 Participants in the Primary Market

View Set